![]() |
|
|
FlashChat | Actuarial Discussion | Preliminary Exams | CAS/SOA Exams | Cyberchat | Around the World | Suggestions |
|
Probability Old Exam P Forum |
![]() |
|
Thread Tools | Search this Thread | Display Modes |
#31
|
||||
|
||||
![]() From #25 of SE4.........I have a question about Uniform...
If given X & Y are uniformly distributed on a region, what about X or Y separately...can we assume that they are both uniform or not??? For #25, I computed it by using f(x,y)=f(x)*f(y)=1/16.......the result is 0.56. So I don't know if I did computation wrong or I used wrong f(x,y) at the beginning??? |
#32
|
||||
|
||||
![]() Quote:
|
#33
|
||||
|
||||
![]() Quote:
Thank you!!! |
#34
|
|||
|
|||
![]() I will be prepping for P exam later this year. Not sure which is best way to prep. Does TIA have written material that can be studied at my pace? Are the video's offered only for a period of time? How does TIA compare ASM? BPP?
|
#36
|
|||
|
|||
![]() Hello. I am wondering if someone could help me out with a few problems such as #4 and #21, #25 respectively on TIA practice exam 2.
Q1: For #4 Why is g(x) monotone for f(x)= 2/(pi(1+4x^2))? To my understanding of monotone, the function must be strictly increasing or decreasing. Not seeing this relationship because the first 3 terms plugging in -1,0,1 for x respectively I get [2/(pi(5)],[2/pi],[2/(pi(5)]). Which is not strictly increasing or decreasing. What am I misunderstanding? Q2:For #21 Considering these random variables X= Cost 1 follows Uniform(0,4) Y= Cost 2 follows Uniform(0,4) in thousands P= Insurance Payment follows continuous distribution P= { X + Y for 0<X+Y<6 , 6 for X+Y>6} where P=min(X+Y,6000) Based on the graph in the video I tried to split it into 2 separate regions above the line and below the line. I: Below the line , II: Above the line This is what I have: E(P)=([E(I)+E(II)]*(1/3)(1/3)) I think we might need two double integrals for the region below the line. Is that correct? So for Region (I) I have: the outer integral from 2 to 4 and 0 to 6-x the inner integral (1/16)(X+Y) dy dx + the outer integral from 0 to 2 and 0 to 4 the inner integral (1/16)(X+Y) dy dx. Region(II) = the outer integral from 6-x to 4 and 2 to 4 the inner integral (1/16)(6) dy dx. Unfortunately I am not coming up with the correct answer perhaps this setup is incorrect? What am I misunderstanding? Q3: For # 25 Why do we want the Var(min(X,Y)) and not Var(X+Y)? Any help would be greatly appreciated. Thank you very much. |
#37
|
|||
|
|||
![]() Quote:
Quote:
Here's the integrations: One part Another part Last bit Now that's the expected payment in the case where both of the machines require maintenance. If only one machine requires maintenance, then payment will cover the whole thing, since that amount won't exceed 6000. So that means we have an additional expected amount of 2000 with probability (1/3)(2/3) (case where the first machine breaks) and another 2000 with probability (2/3)(1/3) (case where just the second machine breaks. The grand total, then is And the difference is that I rounded the 0.435. Quote:
Does that help? |
#39
|
|||
|
|||
![]() Hello I think I am misunderstanding how to organize the payment structure for this problem. I hope someone could help me out.
N= # of Losses ~Poisson(lamda=2.5) Y= Payment Goal : E(Y) I understand there are 2 blocks of payments; first block is a constant 5000; the second block varies; so in this case for the first loss we have (5000(for block 1) and 5000(for block 2) representing the 10000 in total for the first loss, the second loss we have (5000 for (block 1) , 2500 for (block 2))representing the 7500 in total for the second loss etc. Q: So how do we use those terms representing the 2 blocks of payments regarding to the expectation? Any help would be greatly appreciated. Thank you very much. Last edited by RobertR1990; 09-18-2016 at 12:22 PM.. |
#40
|
|||
|
|||
![]() We include the 5000 second block whenever N is greater than 1, since in that case we know that the additional 5000 was paid. We include the 2500 second block when N is at least 2, since we know that in that case the additional 2500 was paid.
Try it directly. Rewrite as an increasing bit and some constant bits: Does that help? |
![]() |
Tags |
the infinite actuary, tia |
Thread Tools | Search this Thread |
Display Modes | |
|
|